A coin, having probability p of landing heads and probability of q=(1-p) of landing on heads. ...

Why did the rest of the Eastern Bloc not invade Yugoslavia?

3 doors, three guards, one stone

Why is "Consequences inflicted." not a sentence?

Why did the IBM 650 use bi-quinary?

How to react to hostile behavior from a senior developer?

List of Python versions

When do you get frequent flier miles - when you buy, or when you fly?

What is the meaning of the new sigil in Game of Thrones Season 8 intro?

prime numbers and expressing non-prime numbers

Single word antonym of "flightless"

Identifying polygons that intersect with another layer using QGIS?

How discoverable are IPv6 addresses and AAAA names by potential attackers?

Why are Kinder Surprise Eggs illegal in the USA?

What exactly is a "Meth" in Altered Carbon?

Withdrew £2800, but only £2000 shows as withdrawn on online banking; what are my obligations?

How can I make names more distinctive without making them longer?

Is it true that "carbohydrates are of no use for the basal metabolic need"?

How to find all the available tools in macOS terminal?

Why is my conclusion inconsistent with the van't Hoff equation?

How to align text above triangle figure

How to bypass password on Windows XP account?

How would the world control an invulnerable immortal mass murderer?

List *all* the tuples!

Is there a (better) way to access $wpdb results?



A coin, having probability p of landing heads and probability of q=(1-p) of landing on heads.



Announcing the arrival of Valued Associate #679: Cesar Manara
Planned maintenance scheduled April 17/18, 2019 at 00:00UTC (8:00pm US/Eastern)Flipping a special coin: probability of getting heads equals the proportion of heads in the flips so farBiased coin flipped until $r$ heads appearBiased coin probabilityCoin-flipping experiment: the expected number of flips that land on headsWhy are odds of a coin landing heads $50%$ after $'n'$ consecutive headsWhat is the probability of a biased coin flipping heads (probability of heads is $frac 35$) exactly $65$ times in $100$ trials?Flipping rigged coin, calculating most common number of flips between headsChernoff bound probability: value of $n$ so that with probability $.999$ at least half of the coin flips come out headsFlip a coin 6 times. Probability with past results and probability without past results are different?Probability density function of flipping until heads and tails












1












$begingroup$


A coin, having probability p of landing heads and probability of q=(1-p) of landing on heads. It is continuously flipped until at least one head and one tail have been flipped.



This is not part of a homework assignment. I am studying for a final and don't understand the professors solutions.



a.) Find the expected number of flips needed.



Since this is clearly geometric, I would think the solution would be:



E(N)=$Sigma_{i=0}^{infty}ip^{n-1}q+Sigma_{i=0}^{n}iq^{n-1}p=frac{1}{q}+frac{1}{p}$.



However, I am completely wrong.
The answer is



E(N)=$p(1+frac{1}{q})+q(1+frac{1}{p})$



For example, consider we flip for heads first. Then we have:



E(N|H)=$p+pSigma_{i=0}^{infty}np^{n-1}q$... I am not sure why this makes sense.



I am not entirely sure why we have an added 1 and a factored p,q. Could someone carefully explain why it makes sense that this is the right answer?










share|cite|improve this question









New contributor




Mistah White is a new contributor to this site. Take care in asking for clarification, commenting, and answering.
Check out our Code of Conduct.







$endgroup$








  • 2




    $begingroup$
    It's all a question of the first toss. If it is $H$ then you just get one more than the expected time to get a $T$, if it is $T$ then you just get one more than the expected time to get $H$. Your method is incorrect because the expected number of tosses needed to get one of the two is $1$.
    $endgroup$
    – lulu
    2 hours ago












  • $begingroup$
    In both the title and first paragraph it appears there is $0$ chance of landing tails, so you will wait forever.
    $endgroup$
    – Ross Millikan
    2 hours ago












  • $begingroup$
    Note: your sums are hard to follow. What's $n$? The upper limit of the sums should be $infty$, the exponent of the probability ought to be a simple function of $i$. Done correctly, your method ought to work (though it's easier to do it the other way).
    $endgroup$
    – lulu
    1 hour ago
















1












$begingroup$


A coin, having probability p of landing heads and probability of q=(1-p) of landing on heads. It is continuously flipped until at least one head and one tail have been flipped.



This is not part of a homework assignment. I am studying for a final and don't understand the professors solutions.



a.) Find the expected number of flips needed.



Since this is clearly geometric, I would think the solution would be:



E(N)=$Sigma_{i=0}^{infty}ip^{n-1}q+Sigma_{i=0}^{n}iq^{n-1}p=frac{1}{q}+frac{1}{p}$.



However, I am completely wrong.
The answer is



E(N)=$p(1+frac{1}{q})+q(1+frac{1}{p})$



For example, consider we flip for heads first. Then we have:



E(N|H)=$p+pSigma_{i=0}^{infty}np^{n-1}q$... I am not sure why this makes sense.



I am not entirely sure why we have an added 1 and a factored p,q. Could someone carefully explain why it makes sense that this is the right answer?










share|cite|improve this question









New contributor




Mistah White is a new contributor to this site. Take care in asking for clarification, commenting, and answering.
Check out our Code of Conduct.







$endgroup$








  • 2




    $begingroup$
    It's all a question of the first toss. If it is $H$ then you just get one more than the expected time to get a $T$, if it is $T$ then you just get one more than the expected time to get $H$. Your method is incorrect because the expected number of tosses needed to get one of the two is $1$.
    $endgroup$
    – lulu
    2 hours ago












  • $begingroup$
    In both the title and first paragraph it appears there is $0$ chance of landing tails, so you will wait forever.
    $endgroup$
    – Ross Millikan
    2 hours ago












  • $begingroup$
    Note: your sums are hard to follow. What's $n$? The upper limit of the sums should be $infty$, the exponent of the probability ought to be a simple function of $i$. Done correctly, your method ought to work (though it's easier to do it the other way).
    $endgroup$
    – lulu
    1 hour ago














1












1








1


1



$begingroup$


A coin, having probability p of landing heads and probability of q=(1-p) of landing on heads. It is continuously flipped until at least one head and one tail have been flipped.



This is not part of a homework assignment. I am studying for a final and don't understand the professors solutions.



a.) Find the expected number of flips needed.



Since this is clearly geometric, I would think the solution would be:



E(N)=$Sigma_{i=0}^{infty}ip^{n-1}q+Sigma_{i=0}^{n}iq^{n-1}p=frac{1}{q}+frac{1}{p}$.



However, I am completely wrong.
The answer is



E(N)=$p(1+frac{1}{q})+q(1+frac{1}{p})$



For example, consider we flip for heads first. Then we have:



E(N|H)=$p+pSigma_{i=0}^{infty}np^{n-1}q$... I am not sure why this makes sense.



I am not entirely sure why we have an added 1 and a factored p,q. Could someone carefully explain why it makes sense that this is the right answer?










share|cite|improve this question









New contributor




Mistah White is a new contributor to this site. Take care in asking for clarification, commenting, and answering.
Check out our Code of Conduct.







$endgroup$




A coin, having probability p of landing heads and probability of q=(1-p) of landing on heads. It is continuously flipped until at least one head and one tail have been flipped.



This is not part of a homework assignment. I am studying for a final and don't understand the professors solutions.



a.) Find the expected number of flips needed.



Since this is clearly geometric, I would think the solution would be:



E(N)=$Sigma_{i=0}^{infty}ip^{n-1}q+Sigma_{i=0}^{n}iq^{n-1}p=frac{1}{q}+frac{1}{p}$.



However, I am completely wrong.
The answer is



E(N)=$p(1+frac{1}{q})+q(1+frac{1}{p})$



For example, consider we flip for heads first. Then we have:



E(N|H)=$p+pSigma_{i=0}^{infty}np^{n-1}q$... I am not sure why this makes sense.



I am not entirely sure why we have an added 1 and a factored p,q. Could someone carefully explain why it makes sense that this is the right answer?







probability probability-theory probability-distributions expected-value






share|cite|improve this question









New contributor




Mistah White is a new contributor to this site. Take care in asking for clarification, commenting, and answering.
Check out our Code of Conduct.











share|cite|improve this question









New contributor




Mistah White is a new contributor to this site. Take care in asking for clarification, commenting, and answering.
Check out our Code of Conduct.









share|cite|improve this question




share|cite|improve this question








edited 1 hour ago







Mistah White













New contributor




Mistah White is a new contributor to this site. Take care in asking for clarification, commenting, and answering.
Check out our Code of Conduct.









asked 2 hours ago









Mistah WhiteMistah White

62




62




New contributor




Mistah White is a new contributor to this site. Take care in asking for clarification, commenting, and answering.
Check out our Code of Conduct.





New contributor





Mistah White is a new contributor to this site. Take care in asking for clarification, commenting, and answering.
Check out our Code of Conduct.






Mistah White is a new contributor to this site. Take care in asking for clarification, commenting, and answering.
Check out our Code of Conduct.








  • 2




    $begingroup$
    It's all a question of the first toss. If it is $H$ then you just get one more than the expected time to get a $T$, if it is $T$ then you just get one more than the expected time to get $H$. Your method is incorrect because the expected number of tosses needed to get one of the two is $1$.
    $endgroup$
    – lulu
    2 hours ago












  • $begingroup$
    In both the title and first paragraph it appears there is $0$ chance of landing tails, so you will wait forever.
    $endgroup$
    – Ross Millikan
    2 hours ago












  • $begingroup$
    Note: your sums are hard to follow. What's $n$? The upper limit of the sums should be $infty$, the exponent of the probability ought to be a simple function of $i$. Done correctly, your method ought to work (though it's easier to do it the other way).
    $endgroup$
    – lulu
    1 hour ago














  • 2




    $begingroup$
    It's all a question of the first toss. If it is $H$ then you just get one more than the expected time to get a $T$, if it is $T$ then you just get one more than the expected time to get $H$. Your method is incorrect because the expected number of tosses needed to get one of the two is $1$.
    $endgroup$
    – lulu
    2 hours ago












  • $begingroup$
    In both the title and first paragraph it appears there is $0$ chance of landing tails, so you will wait forever.
    $endgroup$
    – Ross Millikan
    2 hours ago












  • $begingroup$
    Note: your sums are hard to follow. What's $n$? The upper limit of the sums should be $infty$, the exponent of the probability ought to be a simple function of $i$. Done correctly, your method ought to work (though it's easier to do it the other way).
    $endgroup$
    – lulu
    1 hour ago








2




2




$begingroup$
It's all a question of the first toss. If it is $H$ then you just get one more than the expected time to get a $T$, if it is $T$ then you just get one more than the expected time to get $H$. Your method is incorrect because the expected number of tosses needed to get one of the two is $1$.
$endgroup$
– lulu
2 hours ago






$begingroup$
It's all a question of the first toss. If it is $H$ then you just get one more than the expected time to get a $T$, if it is $T$ then you just get one more than the expected time to get $H$. Your method is incorrect because the expected number of tosses needed to get one of the two is $1$.
$endgroup$
– lulu
2 hours ago














$begingroup$
In both the title and first paragraph it appears there is $0$ chance of landing tails, so you will wait forever.
$endgroup$
– Ross Millikan
2 hours ago






$begingroup$
In both the title and first paragraph it appears there is $0$ chance of landing tails, so you will wait forever.
$endgroup$
– Ross Millikan
2 hours ago














$begingroup$
Note: your sums are hard to follow. What's $n$? The upper limit of the sums should be $infty$, the exponent of the probability ought to be a simple function of $i$. Done correctly, your method ought to work (though it's easier to do it the other way).
$endgroup$
– lulu
1 hour ago




$begingroup$
Note: your sums are hard to follow. What's $n$? The upper limit of the sums should be $infty$, the exponent of the probability ought to be a simple function of $i$. Done correctly, your method ought to work (though it's easier to do it the other way).
$endgroup$
– lulu
1 hour ago










2 Answers
2






active

oldest

votes


















3












$begingroup$

If you get a head with probability $p$ then the expected number of throws is $1+E(X)$ where $X$ is a geometric distribution requiring a tail to be thrown with probability $q$ so $1+E(X)=1+frac1q$. Similarly if you throw a tail with probability $q$ then the expected number of throws is $1+E(Y)$ where $Y$ is a geometric distribution requiring a head to be thrown with probability $p$ so $1+E(Y)=1+frac1p$. This means that the overall expected number of throws is
$$pleft(1+frac1qright)+qleft(1+frac1pright)$$
because there is a probability $p$ that the expected number of throws is given by $1+E(X)$ and probability $q$ that it is given by $1+E(Y)$.






share|cite|improve this answer











$endgroup$





















    2












    $begingroup$

    Let $X$ be the time of the first head, and $Y$ the time of the first tail, and $W$ the first time when a head and a tail has been flipped.



    You are right in assuming that $E[X]=frac{1}{p}$ and $E[Y]=frac{1}{q}$, But you are wrong in assuming that $W=X+Y$, that's simply not true, actually $W=max(X,Y)$.



    A possible approach. Let $A$ be the indicator variable of the event: "first coin was a head" (hence $X=1$).



    Then use $$E[W]=E[E[W | A ]] = P(A=1) E[W|A=1]+P(A=0) E[W|A=0]=\=p(E[Y]+1)+q(E[X]+1)$$






    share|cite|improve this answer











    $endgroup$














      Your Answer








      StackExchange.ready(function() {
      var channelOptions = {
      tags: "".split(" "),
      id: "69"
      };
      initTagRenderer("".split(" "), "".split(" "), channelOptions);

      StackExchange.using("externalEditor", function() {
      // Have to fire editor after snippets, if snippets enabled
      if (StackExchange.settings.snippets.snippetsEnabled) {
      StackExchange.using("snippets", function() {
      createEditor();
      });
      }
      else {
      createEditor();
      }
      });

      function createEditor() {
      StackExchange.prepareEditor({
      heartbeatType: 'answer',
      autoActivateHeartbeat: false,
      convertImagesToLinks: true,
      noModals: true,
      showLowRepImageUploadWarning: true,
      reputationToPostImages: 10,
      bindNavPrevention: true,
      postfix: "",
      imageUploader: {
      brandingHtml: "Powered by u003ca class="icon-imgur-white" href="https://imgur.com/"u003eu003c/au003e",
      contentPolicyHtml: "User contributions licensed under u003ca href="https://creativecommons.org/licenses/by-sa/3.0/"u003ecc by-sa 3.0 with attribution requiredu003c/au003e u003ca href="https://stackoverflow.com/legal/content-policy"u003e(content policy)u003c/au003e",
      allowUrls: true
      },
      noCode: true, onDemand: true,
      discardSelector: ".discard-answer"
      ,immediatelyShowMarkdownHelp:true
      });


      }
      });






      Mistah White is a new contributor. Be nice, and check out our Code of Conduct.










      draft saved

      draft discarded


















      StackExchange.ready(
      function () {
      StackExchange.openid.initPostLogin('.new-post-login', 'https%3a%2f%2fmath.stackexchange.com%2fquestions%2f3190435%2fa-coin-having-probability-p-of-landing-heads-and-probability-of-q-1-p-of-land%23new-answer', 'question_page');
      }
      );

      Post as a guest















      Required, but never shown

























      2 Answers
      2






      active

      oldest

      votes








      2 Answers
      2






      active

      oldest

      votes









      active

      oldest

      votes






      active

      oldest

      votes









      3












      $begingroup$

      If you get a head with probability $p$ then the expected number of throws is $1+E(X)$ where $X$ is a geometric distribution requiring a tail to be thrown with probability $q$ so $1+E(X)=1+frac1q$. Similarly if you throw a tail with probability $q$ then the expected number of throws is $1+E(Y)$ where $Y$ is a geometric distribution requiring a head to be thrown with probability $p$ so $1+E(Y)=1+frac1p$. This means that the overall expected number of throws is
      $$pleft(1+frac1qright)+qleft(1+frac1pright)$$
      because there is a probability $p$ that the expected number of throws is given by $1+E(X)$ and probability $q$ that it is given by $1+E(Y)$.






      share|cite|improve this answer











      $endgroup$


















        3












        $begingroup$

        If you get a head with probability $p$ then the expected number of throws is $1+E(X)$ where $X$ is a geometric distribution requiring a tail to be thrown with probability $q$ so $1+E(X)=1+frac1q$. Similarly if you throw a tail with probability $q$ then the expected number of throws is $1+E(Y)$ where $Y$ is a geometric distribution requiring a head to be thrown with probability $p$ so $1+E(Y)=1+frac1p$. This means that the overall expected number of throws is
        $$pleft(1+frac1qright)+qleft(1+frac1pright)$$
        because there is a probability $p$ that the expected number of throws is given by $1+E(X)$ and probability $q$ that it is given by $1+E(Y)$.






        share|cite|improve this answer











        $endgroup$
















          3












          3








          3





          $begingroup$

          If you get a head with probability $p$ then the expected number of throws is $1+E(X)$ where $X$ is a geometric distribution requiring a tail to be thrown with probability $q$ so $1+E(X)=1+frac1q$. Similarly if you throw a tail with probability $q$ then the expected number of throws is $1+E(Y)$ where $Y$ is a geometric distribution requiring a head to be thrown with probability $p$ so $1+E(Y)=1+frac1p$. This means that the overall expected number of throws is
          $$pleft(1+frac1qright)+qleft(1+frac1pright)$$
          because there is a probability $p$ that the expected number of throws is given by $1+E(X)$ and probability $q$ that it is given by $1+E(Y)$.






          share|cite|improve this answer











          $endgroup$



          If you get a head with probability $p$ then the expected number of throws is $1+E(X)$ where $X$ is a geometric distribution requiring a tail to be thrown with probability $q$ so $1+E(X)=1+frac1q$. Similarly if you throw a tail with probability $q$ then the expected number of throws is $1+E(Y)$ where $Y$ is a geometric distribution requiring a head to be thrown with probability $p$ so $1+E(Y)=1+frac1p$. This means that the overall expected number of throws is
          $$pleft(1+frac1qright)+qleft(1+frac1pright)$$
          because there is a probability $p$ that the expected number of throws is given by $1+E(X)$ and probability $q$ that it is given by $1+E(Y)$.







          share|cite|improve this answer














          share|cite|improve this answer



          share|cite|improve this answer








          edited 1 hour ago

























          answered 1 hour ago









          Peter ForemanPeter Foreman

          7,8751320




          7,8751320























              2












              $begingroup$

              Let $X$ be the time of the first head, and $Y$ the time of the first tail, and $W$ the first time when a head and a tail has been flipped.



              You are right in assuming that $E[X]=frac{1}{p}$ and $E[Y]=frac{1}{q}$, But you are wrong in assuming that $W=X+Y$, that's simply not true, actually $W=max(X,Y)$.



              A possible approach. Let $A$ be the indicator variable of the event: "first coin was a head" (hence $X=1$).



              Then use $$E[W]=E[E[W | A ]] = P(A=1) E[W|A=1]+P(A=0) E[W|A=0]=\=p(E[Y]+1)+q(E[X]+1)$$






              share|cite|improve this answer











              $endgroup$


















                2












                $begingroup$

                Let $X$ be the time of the first head, and $Y$ the time of the first tail, and $W$ the first time when a head and a tail has been flipped.



                You are right in assuming that $E[X]=frac{1}{p}$ and $E[Y]=frac{1}{q}$, But you are wrong in assuming that $W=X+Y$, that's simply not true, actually $W=max(X,Y)$.



                A possible approach. Let $A$ be the indicator variable of the event: "first coin was a head" (hence $X=1$).



                Then use $$E[W]=E[E[W | A ]] = P(A=1) E[W|A=1]+P(A=0) E[W|A=0]=\=p(E[Y]+1)+q(E[X]+1)$$






                share|cite|improve this answer











                $endgroup$
















                  2












                  2








                  2





                  $begingroup$

                  Let $X$ be the time of the first head, and $Y$ the time of the first tail, and $W$ the first time when a head and a tail has been flipped.



                  You are right in assuming that $E[X]=frac{1}{p}$ and $E[Y]=frac{1}{q}$, But you are wrong in assuming that $W=X+Y$, that's simply not true, actually $W=max(X,Y)$.



                  A possible approach. Let $A$ be the indicator variable of the event: "first coin was a head" (hence $X=1$).



                  Then use $$E[W]=E[E[W | A ]] = P(A=1) E[W|A=1]+P(A=0) E[W|A=0]=\=p(E[Y]+1)+q(E[X]+1)$$






                  share|cite|improve this answer











                  $endgroup$



                  Let $X$ be the time of the first head, and $Y$ the time of the first tail, and $W$ the first time when a head and a tail has been flipped.



                  You are right in assuming that $E[X]=frac{1}{p}$ and $E[Y]=frac{1}{q}$, But you are wrong in assuming that $W=X+Y$, that's simply not true, actually $W=max(X,Y)$.



                  A possible approach. Let $A$ be the indicator variable of the event: "first coin was a head" (hence $X=1$).



                  Then use $$E[W]=E[E[W | A ]] = P(A=1) E[W|A=1]+P(A=0) E[W|A=0]=\=p(E[Y]+1)+q(E[X]+1)$$







                  share|cite|improve this answer














                  share|cite|improve this answer



                  share|cite|improve this answer








                  edited 1 hour ago

























                  answered 1 hour ago









                  leonbloyleonbloy

                  42.5k647108




                  42.5k647108






















                      Mistah White is a new contributor. Be nice, and check out our Code of Conduct.










                      draft saved

                      draft discarded


















                      Mistah White is a new contributor. Be nice, and check out our Code of Conduct.













                      Mistah White is a new contributor. Be nice, and check out our Code of Conduct.












                      Mistah White is a new contributor. Be nice, and check out our Code of Conduct.
















                      Thanks for contributing an answer to Mathematics Stack Exchange!


                      • Please be sure to answer the question. Provide details and share your research!

                      But avoid



                      • Asking for help, clarification, or responding to other answers.

                      • Making statements based on opinion; back them up with references or personal experience.


                      Use MathJax to format equations. MathJax reference.


                      To learn more, see our tips on writing great answers.




                      draft saved


                      draft discarded














                      StackExchange.ready(
                      function () {
                      StackExchange.openid.initPostLogin('.new-post-login', 'https%3a%2f%2fmath.stackexchange.com%2fquestions%2f3190435%2fa-coin-having-probability-p-of-landing-heads-and-probability-of-q-1-p-of-land%23new-answer', 'question_page');
                      }
                      );

                      Post as a guest















                      Required, but never shown





















































                      Required, but never shown














                      Required, but never shown












                      Required, but never shown







                      Required, but never shown

































                      Required, but never shown














                      Required, but never shown












                      Required, but never shown







                      Required, but never shown







                      Popular posts from this blog

                      As a Security Precaution, the user account has been locked The Next CEO of Stack OverflowMS...

                      Список ссавців Італії Природоохоронні статуси | Список |...

                      Українські прізвища Зміст Історичні відомості |...